Prove that the area of the triangle formed by the medians is equal to $3/4$ the area of the original triangleHow to find area of triangle from its mediansInequality in triangle involving side lenghs, medians and areaRatio of area of triangle to that formed by its mediansTriangle inscribed inside a circle: prove that $abc = 4 times area times R$Area of a circumcenter triangle equals area of medial triangleA plane contains a set of marked points, such that any three can be covered by a unit disk. Prove that the entire set can be covered by a unit disk.Inequality from IMO 2000 problem 4 question $prodlimits_cycleft(a-1+frac1bright)leq 1$ $abc=1$For all triangle prove that $sumlimits_cycm_acosfracalpha2geqfrac34(a+b+c)$Parallelogram Contest ProblemFinding Value of Expression in a TriangleFinding the area of inner triangle constructed by three cevian lines of a large triangle

What are the threaded holes in Manfrotto camera brackets?

What problems would a superhuman have whose skin is constantly hot?

Plausibility of Mushroom Buildings

Should I take out a loan for a friend to invest on my behalf?

Warn me when the equation number is pushed to the next line

PTIJ: Should I kill my computer after installing software?

How strictly should I take "Candidates must be local"?

Are there historical instances of the capital of a colonising country being temporarily or permanently shifted to one of its colonies?

Was Luke Skywalker the leader of the Rebel forces on Hoth?

Can one live in the U.S. and not use a credit card?

Can I pump my MTB tire to max (55 psi / 380 kPa) without the tube inside bursting?

Vocabulary for giving just numbers, not a full answer

Does the nature of the Apocalypse in The Umbrella Academy change from the first to the last episode?

Reversed Sudoku

How to write ı (i without dot) character in pgf-pie

How can The Temple of Elementary Evil reliably protect itself against kinetic bombardment?

Find longest word in a string: are any of these algorithms good?

Latex does not go to next line

PTIJ: wiping amalek’s memory?

UART pins to unpowered MCU?

Does a warlock using the Darkness/Devil's Sight combo still have advantage on ranged attacks against a target outside the Darkness?

How do I express some one as a black person?

How did Alan Turing break the enigma code using the hint given by the lady in the bar?

NASA's RS-25 Engines shut down time



Prove that the area of the triangle formed by the medians is equal to $3/4$ the area of the original triangle


How to find area of triangle from its mediansInequality in triangle involving side lenghs, medians and areaRatio of area of triangle to that formed by its mediansTriangle inscribed inside a circle: prove that $abc = 4 times area times R$Area of a circumcenter triangle equals area of medial triangleA plane contains a set of marked points, such that any three can be covered by a unit disk. Prove that the entire set can be covered by a unit disk.Inequality from IMO 2000 problem 4 question $prodlimits_cycleft(a-1+frac1bright)leq 1$ $abc=1$For all triangle prove that $sumlimits_cycm_acosfracalpha2geqfrac34(a+b+c)$Parallelogram Contest ProblemFinding Value of Expression in a TriangleFinding the area of inner triangle constructed by three cevian lines of a large triangle













1












$begingroup$


This is Question $2$ from this document on Olympiad Geometry.




Let $ABC$ be a triangle and $M_A,M_B,M_C$ the midpoints of the sides $BC, CA, AB$, respectively. Show that the triangle with side lengths $AM_A, BM_B, CM_C$ has area $3/4$ that of the triangle $ABC$.




This is part of a chapter that stresses that by "slicing and dicing", we can solve a lot of complicated problems. Hence, the stress is on diagrammatic proofs.



To form a triangle with the medians, I extended $AM_A$ beyond $BC$, and formed another copy of the triangle $ABC$. My diagram looks like this:
enter image description here



Obviously $BP=CM_C$. Hence, if $PM_B=AM_A$, we'll have created a triangle with the medians as sides.




So my question is, is $AM_A=PM_B$? A followup question would be is it easy to see that the area of the triangle $BPM_B$ is equal to $3/8$ that of the parallelogram given?











share|cite|improve this question









$endgroup$











  • $begingroup$
    Hint: Yes, $AM_A = PM_B$. Look at triangles $triangle CPM_B$ and $triangle CA'A$. They are similar.
    $endgroup$
    – Sameer Kailasa
    2 days ago










  • $begingroup$
    @SameerKailasa- Haha yes should've seen that. How about proving that the area of $BPM_B$ is equal to $3/8$ the area of $ABCA'$? Is it easy to see from this diagram?
    $endgroup$
    – Anju George
    2 days ago










  • $begingroup$
    You will find very inspiring the answers of math.stackexchange.com/q/396085
    $endgroup$
    – Jean Marie
    2 days ago






  • 1




    $begingroup$
    Besides, use for example geogebra to draw your figures instead of a photograph of an approximate figure drawn on a blackboard :)
    $endgroup$
    – Jean Marie
    2 days ago










  • $begingroup$
    See as well jwilson.coe.uga.edu/emt725/Medians.Triangle/…
    $endgroup$
    – Jean Marie
    yesterday















1












$begingroup$


This is Question $2$ from this document on Olympiad Geometry.




Let $ABC$ be a triangle and $M_A,M_B,M_C$ the midpoints of the sides $BC, CA, AB$, respectively. Show that the triangle with side lengths $AM_A, BM_B, CM_C$ has area $3/4$ that of the triangle $ABC$.




This is part of a chapter that stresses that by "slicing and dicing", we can solve a lot of complicated problems. Hence, the stress is on diagrammatic proofs.



To form a triangle with the medians, I extended $AM_A$ beyond $BC$, and formed another copy of the triangle $ABC$. My diagram looks like this:
enter image description here



Obviously $BP=CM_C$. Hence, if $PM_B=AM_A$, we'll have created a triangle with the medians as sides.




So my question is, is $AM_A=PM_B$? A followup question would be is it easy to see that the area of the triangle $BPM_B$ is equal to $3/8$ that of the parallelogram given?











share|cite|improve this question









$endgroup$











  • $begingroup$
    Hint: Yes, $AM_A = PM_B$. Look at triangles $triangle CPM_B$ and $triangle CA'A$. They are similar.
    $endgroup$
    – Sameer Kailasa
    2 days ago










  • $begingroup$
    @SameerKailasa- Haha yes should've seen that. How about proving that the area of $BPM_B$ is equal to $3/8$ the area of $ABCA'$? Is it easy to see from this diagram?
    $endgroup$
    – Anju George
    2 days ago










  • $begingroup$
    You will find very inspiring the answers of math.stackexchange.com/q/396085
    $endgroup$
    – Jean Marie
    2 days ago






  • 1




    $begingroup$
    Besides, use for example geogebra to draw your figures instead of a photograph of an approximate figure drawn on a blackboard :)
    $endgroup$
    – Jean Marie
    2 days ago










  • $begingroup$
    See as well jwilson.coe.uga.edu/emt725/Medians.Triangle/…
    $endgroup$
    – Jean Marie
    yesterday













1












1








1





$begingroup$


This is Question $2$ from this document on Olympiad Geometry.




Let $ABC$ be a triangle and $M_A,M_B,M_C$ the midpoints of the sides $BC, CA, AB$, respectively. Show that the triangle with side lengths $AM_A, BM_B, CM_C$ has area $3/4$ that of the triangle $ABC$.




This is part of a chapter that stresses that by "slicing and dicing", we can solve a lot of complicated problems. Hence, the stress is on diagrammatic proofs.



To form a triangle with the medians, I extended $AM_A$ beyond $BC$, and formed another copy of the triangle $ABC$. My diagram looks like this:
enter image description here



Obviously $BP=CM_C$. Hence, if $PM_B=AM_A$, we'll have created a triangle with the medians as sides.




So my question is, is $AM_A=PM_B$? A followup question would be is it easy to see that the area of the triangle $BPM_B$ is equal to $3/8$ that of the parallelogram given?











share|cite|improve this question









$endgroup$




This is Question $2$ from this document on Olympiad Geometry.




Let $ABC$ be a triangle and $M_A,M_B,M_C$ the midpoints of the sides $BC, CA, AB$, respectively. Show that the triangle with side lengths $AM_A, BM_B, CM_C$ has area $3/4$ that of the triangle $ABC$.




This is part of a chapter that stresses that by "slicing and dicing", we can solve a lot of complicated problems. Hence, the stress is on diagrammatic proofs.



To form a triangle with the medians, I extended $AM_A$ beyond $BC$, and formed another copy of the triangle $ABC$. My diagram looks like this:
enter image description here



Obviously $BP=CM_C$. Hence, if $PM_B=AM_A$, we'll have created a triangle with the medians as sides.




So my question is, is $AM_A=PM_B$? A followup question would be is it easy to see that the area of the triangle $BPM_B$ is equal to $3/8$ that of the parallelogram given?








contest-math






share|cite|improve this question













share|cite|improve this question











share|cite|improve this question




share|cite|improve this question










asked 2 days ago









Anju GeorgeAnju George

564




564











  • $begingroup$
    Hint: Yes, $AM_A = PM_B$. Look at triangles $triangle CPM_B$ and $triangle CA'A$. They are similar.
    $endgroup$
    – Sameer Kailasa
    2 days ago










  • $begingroup$
    @SameerKailasa- Haha yes should've seen that. How about proving that the area of $BPM_B$ is equal to $3/8$ the area of $ABCA'$? Is it easy to see from this diagram?
    $endgroup$
    – Anju George
    2 days ago










  • $begingroup$
    You will find very inspiring the answers of math.stackexchange.com/q/396085
    $endgroup$
    – Jean Marie
    2 days ago






  • 1




    $begingroup$
    Besides, use for example geogebra to draw your figures instead of a photograph of an approximate figure drawn on a blackboard :)
    $endgroup$
    – Jean Marie
    2 days ago










  • $begingroup$
    See as well jwilson.coe.uga.edu/emt725/Medians.Triangle/…
    $endgroup$
    – Jean Marie
    yesterday
















  • $begingroup$
    Hint: Yes, $AM_A = PM_B$. Look at triangles $triangle CPM_B$ and $triangle CA'A$. They are similar.
    $endgroup$
    – Sameer Kailasa
    2 days ago










  • $begingroup$
    @SameerKailasa- Haha yes should've seen that. How about proving that the area of $BPM_B$ is equal to $3/8$ the area of $ABCA'$? Is it easy to see from this diagram?
    $endgroup$
    – Anju George
    2 days ago










  • $begingroup$
    You will find very inspiring the answers of math.stackexchange.com/q/396085
    $endgroup$
    – Jean Marie
    2 days ago






  • 1




    $begingroup$
    Besides, use for example geogebra to draw your figures instead of a photograph of an approximate figure drawn on a blackboard :)
    $endgroup$
    – Jean Marie
    2 days ago










  • $begingroup$
    See as well jwilson.coe.uga.edu/emt725/Medians.Triangle/…
    $endgroup$
    – Jean Marie
    yesterday















$begingroup$
Hint: Yes, $AM_A = PM_B$. Look at triangles $triangle CPM_B$ and $triangle CA'A$. They are similar.
$endgroup$
– Sameer Kailasa
2 days ago




$begingroup$
Hint: Yes, $AM_A = PM_B$. Look at triangles $triangle CPM_B$ and $triangle CA'A$. They are similar.
$endgroup$
– Sameer Kailasa
2 days ago












$begingroup$
@SameerKailasa- Haha yes should've seen that. How about proving that the area of $BPM_B$ is equal to $3/8$ the area of $ABCA'$? Is it easy to see from this diagram?
$endgroup$
– Anju George
2 days ago




$begingroup$
@SameerKailasa- Haha yes should've seen that. How about proving that the area of $BPM_B$ is equal to $3/8$ the area of $ABCA'$? Is it easy to see from this diagram?
$endgroup$
– Anju George
2 days ago












$begingroup$
You will find very inspiring the answers of math.stackexchange.com/q/396085
$endgroup$
– Jean Marie
2 days ago




$begingroup$
You will find very inspiring the answers of math.stackexchange.com/q/396085
$endgroup$
– Jean Marie
2 days ago




1




1




$begingroup$
Besides, use for example geogebra to draw your figures instead of a photograph of an approximate figure drawn on a blackboard :)
$endgroup$
– Jean Marie
2 days ago




$begingroup$
Besides, use for example geogebra to draw your figures instead of a photograph of an approximate figure drawn on a blackboard :)
$endgroup$
– Jean Marie
2 days ago












$begingroup$
See as well jwilson.coe.uga.edu/emt725/Medians.Triangle/…
$endgroup$
– Jean Marie
yesterday




$begingroup$
See as well jwilson.coe.uga.edu/emt725/Medians.Triangle/…
$endgroup$
– Jean Marie
yesterday










1 Answer
1






active

oldest

votes


















2












$begingroup$

Regarding the question, whether $AM_A=PM_B$, the answer is YES.



Simply observe that $$fracCPCA'=fracCM_BCA$$ Thus, in virtue of the Intercept theorem (also known as "Thales' Theorem") $$fracPM_BAA'=fracCPCA'=frac12$$



Can you end it now?




Alternatively, here you have a proof (almost) without words.
enter image description here



Definition: $[...]$ denotes the area of the polygon "..."



Observe that $CE$ is a median in $Delta CDH$. Thus




$$1=fracDKKHfrac[DKE][EKH]=frac[DKC][KHC]=frac[DKC]-[DKE][KHC]-[EKH]=frac[DEC][EHC]iff [DEC]=[EHC]$$ Similarly $$[DEC]=[DHE]=[EHC]$$




Futhermore $$[DEC]=frac[ABC]4$$ Can you end it now?






share|cite|improve this answer











$endgroup$








  • 1




    $begingroup$
    Nice presentation. A little remark : "Intercept theorem" has another very common name in many countries which is "Thales theorem"
    $endgroup$
    – Jean Marie
    yesterday






  • 1




    $begingroup$
    In Spain we also know it as "Thales Theorem", but when I looked it up in the English Wikipedia, I fount "intercepting theorem"...- Edited!
    $endgroup$
    – Dr. Mathva
    yesterday







  • 2




    $begingroup$
    Thanks for the manes of this mathematician !
    $endgroup$
    – Jean Marie
    yesterday










  • $begingroup$
    +1 @Dr.Mathva ...
    $endgroup$
    – Maria Mazur
    yesterday










Your Answer





StackExchange.ifUsing("editor", function ()
return StackExchange.using("mathjaxEditing", function ()
StackExchange.MarkdownEditor.creationCallbacks.add(function (editor, postfix)
StackExchange.mathjaxEditing.prepareWmdForMathJax(editor, postfix, [["$", "$"], ["\\(","\\)"]]);
);
);
, "mathjax-editing");

StackExchange.ready(function()
var channelOptions =
tags: "".split(" "),
id: "69"
;
initTagRenderer("".split(" "), "".split(" "), channelOptions);

StackExchange.using("externalEditor", function()
// Have to fire editor after snippets, if snippets enabled
if (StackExchange.settings.snippets.snippetsEnabled)
StackExchange.using("snippets", function()
createEditor();
);

else
createEditor();

);

function createEditor()
StackExchange.prepareEditor(
heartbeatType: 'answer',
autoActivateHeartbeat: false,
convertImagesToLinks: true,
noModals: true,
showLowRepImageUploadWarning: true,
reputationToPostImages: 10,
bindNavPrevention: true,
postfix: "",
imageUploader:
brandingHtml: "Powered by u003ca class="icon-imgur-white" href="https://imgur.com/"u003eu003c/au003e",
contentPolicyHtml: "User contributions licensed under u003ca href="https://creativecommons.org/licenses/by-sa/3.0/"u003ecc by-sa 3.0 with attribution requiredu003c/au003e u003ca href="https://stackoverflow.com/legal/content-policy"u003e(content policy)u003c/au003e",
allowUrls: true
,
noCode: true, onDemand: true,
discardSelector: ".discard-answer"
,immediatelyShowMarkdownHelp:true
);



);













draft saved

draft discarded


















StackExchange.ready(
function ()
StackExchange.openid.initPostLogin('.new-post-login', 'https%3a%2f%2fmath.stackexchange.com%2fquestions%2f3141714%2fprove-that-the-area-of-the-triangle-formed-by-the-medians-is-equal-to-3-4-the%23new-answer', 'question_page');

);

Post as a guest















Required, but never shown

























1 Answer
1






active

oldest

votes








1 Answer
1






active

oldest

votes









active

oldest

votes






active

oldest

votes









2












$begingroup$

Regarding the question, whether $AM_A=PM_B$, the answer is YES.



Simply observe that $$fracCPCA'=fracCM_BCA$$ Thus, in virtue of the Intercept theorem (also known as "Thales' Theorem") $$fracPM_BAA'=fracCPCA'=frac12$$



Can you end it now?




Alternatively, here you have a proof (almost) without words.
enter image description here



Definition: $[...]$ denotes the area of the polygon "..."



Observe that $CE$ is a median in $Delta CDH$. Thus




$$1=fracDKKHfrac[DKE][EKH]=frac[DKC][KHC]=frac[DKC]-[DKE][KHC]-[EKH]=frac[DEC][EHC]iff [DEC]=[EHC]$$ Similarly $$[DEC]=[DHE]=[EHC]$$




Futhermore $$[DEC]=frac[ABC]4$$ Can you end it now?






share|cite|improve this answer











$endgroup$








  • 1




    $begingroup$
    Nice presentation. A little remark : "Intercept theorem" has another very common name in many countries which is "Thales theorem"
    $endgroup$
    – Jean Marie
    yesterday






  • 1




    $begingroup$
    In Spain we also know it as "Thales Theorem", but when I looked it up in the English Wikipedia, I fount "intercepting theorem"...- Edited!
    $endgroup$
    – Dr. Mathva
    yesterday







  • 2




    $begingroup$
    Thanks for the manes of this mathematician !
    $endgroup$
    – Jean Marie
    yesterday










  • $begingroup$
    +1 @Dr.Mathva ...
    $endgroup$
    – Maria Mazur
    yesterday















2












$begingroup$

Regarding the question, whether $AM_A=PM_B$, the answer is YES.



Simply observe that $$fracCPCA'=fracCM_BCA$$ Thus, in virtue of the Intercept theorem (also known as "Thales' Theorem") $$fracPM_BAA'=fracCPCA'=frac12$$



Can you end it now?




Alternatively, here you have a proof (almost) without words.
enter image description here



Definition: $[...]$ denotes the area of the polygon "..."



Observe that $CE$ is a median in $Delta CDH$. Thus




$$1=fracDKKHfrac[DKE][EKH]=frac[DKC][KHC]=frac[DKC]-[DKE][KHC]-[EKH]=frac[DEC][EHC]iff [DEC]=[EHC]$$ Similarly $$[DEC]=[DHE]=[EHC]$$




Futhermore $$[DEC]=frac[ABC]4$$ Can you end it now?






share|cite|improve this answer











$endgroup$








  • 1




    $begingroup$
    Nice presentation. A little remark : "Intercept theorem" has another very common name in many countries which is "Thales theorem"
    $endgroup$
    – Jean Marie
    yesterday






  • 1




    $begingroup$
    In Spain we also know it as "Thales Theorem", but when I looked it up in the English Wikipedia, I fount "intercepting theorem"...- Edited!
    $endgroup$
    – Dr. Mathva
    yesterday







  • 2




    $begingroup$
    Thanks for the manes of this mathematician !
    $endgroup$
    – Jean Marie
    yesterday










  • $begingroup$
    +1 @Dr.Mathva ...
    $endgroup$
    – Maria Mazur
    yesterday













2












2








2





$begingroup$

Regarding the question, whether $AM_A=PM_B$, the answer is YES.



Simply observe that $$fracCPCA'=fracCM_BCA$$ Thus, in virtue of the Intercept theorem (also known as "Thales' Theorem") $$fracPM_BAA'=fracCPCA'=frac12$$



Can you end it now?




Alternatively, here you have a proof (almost) without words.
enter image description here



Definition: $[...]$ denotes the area of the polygon "..."



Observe that $CE$ is a median in $Delta CDH$. Thus




$$1=fracDKKHfrac[DKE][EKH]=frac[DKC][KHC]=frac[DKC]-[DKE][KHC]-[EKH]=frac[DEC][EHC]iff [DEC]=[EHC]$$ Similarly $$[DEC]=[DHE]=[EHC]$$




Futhermore $$[DEC]=frac[ABC]4$$ Can you end it now?






share|cite|improve this answer











$endgroup$



Regarding the question, whether $AM_A=PM_B$, the answer is YES.



Simply observe that $$fracCPCA'=fracCM_BCA$$ Thus, in virtue of the Intercept theorem (also known as "Thales' Theorem") $$fracPM_BAA'=fracCPCA'=frac12$$



Can you end it now?




Alternatively, here you have a proof (almost) without words.
enter image description here



Definition: $[...]$ denotes the area of the polygon "..."



Observe that $CE$ is a median in $Delta CDH$. Thus




$$1=fracDKKHfrac[DKE][EKH]=frac[DKC][KHC]=frac[DKC]-[DKE][KHC]-[EKH]=frac[DEC][EHC]iff [DEC]=[EHC]$$ Similarly $$[DEC]=[DHE]=[EHC]$$




Futhermore $$[DEC]=frac[ABC]4$$ Can you end it now?







share|cite|improve this answer














share|cite|improve this answer



share|cite|improve this answer








edited yesterday

























answered yesterday









Dr. MathvaDr. Mathva

2,296526




2,296526







  • 1




    $begingroup$
    Nice presentation. A little remark : "Intercept theorem" has another very common name in many countries which is "Thales theorem"
    $endgroup$
    – Jean Marie
    yesterday






  • 1




    $begingroup$
    In Spain we also know it as "Thales Theorem", but when I looked it up in the English Wikipedia, I fount "intercepting theorem"...- Edited!
    $endgroup$
    – Dr. Mathva
    yesterday







  • 2




    $begingroup$
    Thanks for the manes of this mathematician !
    $endgroup$
    – Jean Marie
    yesterday










  • $begingroup$
    +1 @Dr.Mathva ...
    $endgroup$
    – Maria Mazur
    yesterday












  • 1




    $begingroup$
    Nice presentation. A little remark : "Intercept theorem" has another very common name in many countries which is "Thales theorem"
    $endgroup$
    – Jean Marie
    yesterday






  • 1




    $begingroup$
    In Spain we also know it as "Thales Theorem", but when I looked it up in the English Wikipedia, I fount "intercepting theorem"...- Edited!
    $endgroup$
    – Dr. Mathva
    yesterday







  • 2




    $begingroup$
    Thanks for the manes of this mathematician !
    $endgroup$
    – Jean Marie
    yesterday










  • $begingroup$
    +1 @Dr.Mathva ...
    $endgroup$
    – Maria Mazur
    yesterday







1




1




$begingroup$
Nice presentation. A little remark : "Intercept theorem" has another very common name in many countries which is "Thales theorem"
$endgroup$
– Jean Marie
yesterday




$begingroup$
Nice presentation. A little remark : "Intercept theorem" has another very common name in many countries which is "Thales theorem"
$endgroup$
– Jean Marie
yesterday




1




1




$begingroup$
In Spain we also know it as "Thales Theorem", but when I looked it up in the English Wikipedia, I fount "intercepting theorem"...- Edited!
$endgroup$
– Dr. Mathva
yesterday





$begingroup$
In Spain we also know it as "Thales Theorem", but when I looked it up in the English Wikipedia, I fount "intercepting theorem"...- Edited!
$endgroup$
– Dr. Mathva
yesterday





2




2




$begingroup$
Thanks for the manes of this mathematician !
$endgroup$
– Jean Marie
yesterday




$begingroup$
Thanks for the manes of this mathematician !
$endgroup$
– Jean Marie
yesterday












$begingroup$
+1 @Dr.Mathva ...
$endgroup$
– Maria Mazur
yesterday




$begingroup$
+1 @Dr.Mathva ...
$endgroup$
– Maria Mazur
yesterday

















draft saved

draft discarded
















































Thanks for contributing an answer to Mathematics Stack Exchange!


  • Please be sure to answer the question. Provide details and share your research!

But avoid


  • Asking for help, clarification, or responding to other answers.

  • Making statements based on opinion; back them up with references or personal experience.

Use MathJax to format equations. MathJax reference.


To learn more, see our tips on writing great answers.




draft saved


draft discarded














StackExchange.ready(
function ()
StackExchange.openid.initPostLogin('.new-post-login', 'https%3a%2f%2fmath.stackexchange.com%2fquestions%2f3141714%2fprove-that-the-area-of-the-triangle-formed-by-the-medians-is-equal-to-3-4-the%23new-answer', 'question_page');

);

Post as a guest















Required, but never shown





















































Required, but never shown














Required, but never shown












Required, but never shown







Required, but never shown

































Required, but never shown














Required, but never shown












Required, but never shown







Required, but never shown







Popular posts from this blog

Lowndes Grove History Architecture References Navigation menu32°48′6″N 79°57′58″W / 32.80167°N 79.96611°W / 32.80167; -79.9661132°48′6″N 79°57′58″W / 32.80167°N 79.96611°W / 32.80167; -79.9661178002500"National Register Information System"Historic houses of South Carolina"Lowndes Grove""+32° 48' 6.00", −79° 57' 58.00""Lowndes Grove, Charleston County (260 St. Margaret St., Charleston)""Lowndes Grove"The Charleston ExpositionIt Happened in South Carolina"Lowndes Grove (House), Saint Margaret Street & Sixth Avenue, Charleston, Charleston County, SC(Photographs)"Plantations of the Carolina Low Countrye

random experiment with two different functions on unit interval Announcing the arrival of Valued Associate #679: Cesar Manara Planned maintenance scheduled April 23, 2019 at 00:00UTC (8:00pm US/Eastern)Random variable and probability space notionsRandom Walk with EdgesFinding functions where the increase over a random interval is Poisson distributedNumber of days until dayCan an observed event in fact be of zero probability?Unit random processmodels of coins and uniform distributionHow to get the number of successes given $n$ trials , probability $P$ and a random variable $X$Absorbing Markov chain in a computer. Is “almost every” turned into always convergence in computer executions?Stopped random walk is not uniformly integrable

How should I support this large drywall patch? Planned maintenance scheduled April 23, 2019 at 00:00UTC (8:00pm US/Eastern) Announcing the arrival of Valued Associate #679: Cesar Manara Unicorn Meta Zoo #1: Why another podcast?How do I cover large gaps in drywall?How do I keep drywall around a patch from crumbling?Can I glue a second layer of drywall?How to patch long strip on drywall?Large drywall patch: how to avoid bulging seams?Drywall Mesh Patch vs. Bulge? To remove or not to remove?How to fix this drywall job?Prep drywall before backsplashWhat's the best way to fix this horrible drywall patch job?Drywall patching using 3M Patch Plus Primer